Khoa học máy tính lý thuyết

Q & A cho các nhà khoa học và nhà nghiên cứu máy tính lý thuyết trong các lĩnh vực liên quan



3
Xác minh lượng tử một chiều
Lý thuyết tính toán trạng thái cụm được thiết lập tốt cho đến nay, cho thấy rằng bất kỳ mạch BQP nào cũng có thể được sửa đổi nên nó chỉ sử dụng các cổng lượng tử qubit duy nhất, có thể được kiểm soát theo cách cổ điển, cung …

2
Có thể sử dụng các hạn chế ngẫu nhiên để đạt được giới hạn thấp hơn cho
Có một số kết quả giới hạn kích thước mạch nổi tiếng dựa trên các hạn chế ngẫu nhiên và Bổ đề chuyển đổi .A C0AC0\mathsf{AC^0} Chúng ta có thể phát triển kết quả Bổ đề chuyển đổi để chứng minh kích thước giới hạn dưới cho các mạch T …




2
Sai lệch của đa thức ngẫu nhiên với mức độ thấp so với GF (2) là gì?
Tôi có một câu hỏi liên quan đến đa thức bậc thấp và xác suất: Xác suất (hành vi giả định của) xác suất là một đa thức * ngẫu nhiên, , trên GF (2), với các biến bậc và n có .ppp≤d≤d\le dbias(p)≜|Prx∈{0,1}n(p(x)=0)−Prx∈{0,1}n(p(x)=1)|>ϵbias(p)≜|Prx∈{0,1}n(p(x)=0)−Prx∈{0,1}n(p(x)=1)|>ϵbias(p) \triangleq |\Pr_{x\in\{0,1\}^n}(p(x)=0)-\Pr_{x\in\{0,1\}^n}(p(x)=1)| \gt \epsilon * Khi …





1
Bài toán vectơ thuật toán
Tôi có một vấn đề đại số liên quan đến vectơ trong trường GF (2). Hãy là (0,1) -vectors của chiều n , và m = n O ( 1 ) . Tìm một thuật toán thời gian đa thức đó tìm thấy một (0,1) -vector u của kích thước …



Khi sử dụng trang web của chúng tôi, bạn xác nhận rằng bạn đã đọc và hiểu Chính sách cookieChính sách bảo mật của chúng tôi.
Licensed under cc by-sa 3.0 with attribution required.